1
$\begingroup$

Let $y\in\mathbb{R}^m$, $\tau\in\mathbb{R}$ and $X\in\mathbb{R}^{m\times n}$, with $\tau>0$

I would like to efficiently solve the following problem:


Problem 1

Choose $\alpha,z\in\mathbb{R}^m,\beta\in\mathbb{R}^n$ to minimize: $$(y-\alpha)^\top (y-\alpha) + \tau \beta^\top \beta$$ subject to the constraints that: $$z=X\beta$$ $$\beta^\top 1_n = 1$$ $$\beta\ge 0$$ $$\forall i,j\in\{1,\dots,m\}, z_i\le z_j \rightarrow \alpha_i \le \alpha_j$$


(Here $1_n\in\mathbb{R}^n$ is a vector of ones.)

The final constraint is equivalent to:

$$\forall i,j\in\{1,\dots,m\}, (z_j-z_i,\alpha_j-\alpha_i)\in\left\{(c,d)\in\mathbb{R}^2\middle|c\le 0 \vee d\ge 0\right\},$$

which is clearly non-convex. While the problem can be given a mixed integer quadratic programming formulation, this is unlikely to be computationally feasible.

However, if we knew $z=\hat z$, Problem 1 reduces to:


Problem 2

Choose $\alpha\in\mathbb{R}^m$ to minimize: $$(y-\alpha)^\top (y-\alpha)$$ subject to the constraints that: $$\forall i,j\in\{1,\dots,m\}, \hat z_i\le \hat z_j \rightarrow \alpha_i \le \alpha_j$$


This is the isotonic regression problem, and may be solved very efficiently by the pooled adjacent violators algorithm.

Likewise, if we knew $\alpha=\hat\alpha$, then Problem 1 reduces to:


Problem 3

Choose $z\in\mathbb{R}^m,\beta\in\mathbb{R}^n$ to minimize: $$\beta^\top \beta$$ subject to the constraints that: $$z=X\beta$$ $$\beta^\top 1_n = 1$$ $$\beta\ge 0$$ $$\forall i,j\in\{1,\dots,m\}, \hat\alpha_i > \hat\alpha_j \rightarrow z_i > z_j$$


This is a simple quadratic programming problem (at least once the strict inequality on $z$ is replaced by a weak one with a small margin).

Question

My question is whether Problem 2 or Problem 3 can be exploited to give a computationally feasible (iterative?) algorithm for Problem 1. I would of course also be interested in any other approach to efficiently solving Problem 1.

Note that the naïve algorithm of alternating between solving Problem 2 and solving Problem 3 cannot possibly converge to a solution of Problem 1, as neither Problem 2 nor 3 depend on $\tau$.

$\endgroup$
2

0

You must log in to answer this question.

Start asking to get answers

Find the answer to your question by asking.

Ask question

Explore related questions

See similar questions with these tags.